LSAT and Law School Admissions Forum

Get expert LSAT preparation and law school admissions advice from PowerScore Test Preparation.

User avatar
 Dave Killoran
PowerScore Staff
  • PowerScore Staff
  • Posts: 5853
  • Joined: Mar 25, 2011
|
#84900
Complete Question Explanation
(The complete setup for this game can be found here: lsat/viewtopic.php?t=8333)

The correct answer choice is (C)

The first rule eliminates H and J from being first. The second rule eliminates T from being first. The third rule does not eliminate any artifact from being first. Thus, three artifacts cannot be first (H, J, T), leaving three that can be first (F, N, P). Answer choice (C) is thus correct.

Get the most out of your LSAT Prep Plus subscription.

Analyze and track your performance with our Testing and Analytics Package.